- Mon Nov 06, 2017 4:51 pm
#41254
Complete Question Explanation
(The complete setup for this game can be found here: lsat/viewtopic.php?t=6505)
The correct answer choice is (A)
The question stem asks for an answer that could provide a single viable solution for the representatives in Zone 3 (and not for a list of every representative that could conceivably work in Zone 3).
Answer choice (A) is the correct answer. In Template #2, K and M could be the only two representatives working in Zone 3 (P, Q, S, and U would work in Zone 1, and T would work in Zone 2).
Answer choice (B) is incorrect because the only template where T works in Zone 3 is Template #3, and in that template M must also work in Zone 3.
Answer choice (C) is incorrect because no scenario exists where only P and Q work in Zone 3. In Template #1—the only template with P and Q working in Zone 3—at the minimum K or M must also work in Zone 3.
Answer choice (D) is incorrect because by placing both T and U in Zone 3, no representative remains to fulfill the requirements of the second rule.
Answer choice (E) is incorrect because by placing both P and U in Zone 3, only T would remain to fulfill the requirements of the first two rules. Because one representative cannot simultaneously work in two zones, this answer choice is therefore incorrect.
(The complete setup for this game can be found here: lsat/viewtopic.php?t=6505)
The correct answer choice is (A)
The question stem asks for an answer that could provide a single viable solution for the representatives in Zone 3 (and not for a list of every representative that could conceivably work in Zone 3).
Answer choice (A) is the correct answer. In Template #2, K and M could be the only two representatives working in Zone 3 (P, Q, S, and U would work in Zone 1, and T would work in Zone 2).
Answer choice (B) is incorrect because the only template where T works in Zone 3 is Template #3, and in that template M must also work in Zone 3.
Answer choice (C) is incorrect because no scenario exists where only P and Q work in Zone 3. In Template #1—the only template with P and Q working in Zone 3—at the minimum K or M must also work in Zone 3.
Answer choice (D) is incorrect because by placing both T and U in Zone 3, no representative remains to fulfill the requirements of the second rule.
Answer choice (E) is incorrect because by placing both P and U in Zone 3, only T would remain to fulfill the requirements of the first two rules. Because one representative cannot simultaneously work in two zones, this answer choice is therefore incorrect.